Đến nội dung

toanND nội dung

Có 50 mục bởi toanND (Tìm giới hạn từ 27-04-2020)



Sắp theo                Sắp xếp  

#728967 IJ đi qua điểm chính giữa cung BC

Đã gửi bởi toanND on 17-07-2021 - 19:08 trong Hình học

Lời giải của mình: 

Gọi $E$ là giao điểm của $AC$ $BD$. $F$ là giao điểm của $AB$ $CD$.

Gọi $L$ là điểm chính giữa cung $BC$ không chứa $A,D$, $LI$ cắt $(O)$ lần thứ hai tại $T$.

Gọi $X,Y$ lần lượt là tâm các đường tròn bàng tiếp góc $C,B$ của $\triangle ABC, \triangle BDC$

Nếu gọi $S$ là tâm đường tròn nội tiếp tam giác $ABC$ thì ta có $\angle XBS = \angle XAS = 90^0 \Rightarrow XBSA$ nội tiếp

$\Rightarrow \angle BXI = \angle BAS = \angle BTI \Rightarrow$ Tứ giác $XBIT$ nội tiếp.

Tương tự ta cũng chứng minh được tứ giác $TICY$ nội tiếp.

Từ đó ta có $\angle BXC = \angle BTL = \angle CTL = \angle BYC \Rightarrow XBCY$ nội tiếp.

Do $XB, BJ$ đều là phân giác ngoài góc $ABC$ nên $X,B,J$ thẳng hàng. Tương tự, $Y,C,J$ thẳng hàng.

Theo tính chất trục đẳng phương thì trục đẳng phương của ba đường tròn xanh phải đồng quy, tức là $XB, TI, YC$ đồng quy. Mà $XB$ cắt $YC$ tại $J$, suy ra $T,I,J$ thẳng thàng.

Do đó $I,J,L$ thẳng hàng hay ta có $đpcm \blacksquare$ 

comeback 1.PNG

P/S: Nếu ta dựng một đường tròn $\omega$ tiếp xúc với $AE, DE$ và tiếp xúc trong với $(O)$ thì $T$ là tiếp điểm của $\omega$ với $(O)$

comback 2.PNG




#724550 CMR $ TM // BC $

Đã gửi bởi toanND on 08-08-2019 - 09:38 trong Hình học

Bài này có thể giải bằng định lý Pascal




#724490 $\widehat{NLP}=90^0$

Đã gửi bởi toanND on 05-08-2019 - 21:44 trong Hình học

capture NVL.PNG

Gọi AQ là đường đối trung ứng với đỉnh A của tam giác ABC. AQ cắt (O) lần thứ hai tại T

$\Rightarrow$ Tứ giác ABTC điều hòa $\Rightarrow$ SA tiếp xúc với (O) tại A (S là giao của OL với BC).

Gọi $H_{a}$ là chân đường cao kẻ từ A của tam giác ABC. K là giao của OL với $AH_{a}$.

$\Rightarrow$ K là trực tâm tam giác AQS $\Rightarrow QK \perp AS$

Gọi M, I lần lượt là trung điểm của BC, $H_{b}H_{c}$. Khi đó M, I, N thẳng hàng và $MN\perp H_{b}H_{c}$

Mặt khác dễ thấy $AS \parallel H_bH_c$ $\Rightarrow QK \parallel MN$

$KLQH_a$ nội tiếp $\Rightarrow \widehat{H_aLQ}=\widehat{H_aKQ}=\widehat{H_aNI}(KQ\parallel MN)$

$\Rightarrow NILH_a$ nội tiếp. Mặt khác $I,H_a$ cùng nằm trên đường tròn đường kính NP

Suy ra L nằm trên đường tròn đường kính NP$\Rightarrow \widehat{NLP}=90^0$ (đpcm)  :like




#724419 $1+\sqrt{\frac{2+1}{2}}+\sq...

Đã gửi bởi toanND on 01-08-2019 - 21:05 trong Bất đẳng thức và cực trị

 chỗ $\left ( 1+\frac{1}{n^{2}} \right )^{n}> 1+n.\frac{1}{n^{2}}$ này là sao bạn

Bất đẳng thức Bernoulli đó




#723712 Bất đẳng thức

Đã gửi bởi toanND on 13-07-2019 - 09:53 trong Bất đẳng thức và cực trị

  • Trước hết, ta chứng minh bđt phụ sau:

Cho các số thực dương a,b,c. Chứng minh rằng $\frac{a}{c}+\frac{c}{b}+\frac{b}{a}\geq\frac{\sqrt{3(a^2+b^2+c^2)}}{\sqrt[3]{abc}}$ (1)

Thật vậy $(1)\Leftrightarrow \frac{a^2}{c^2}+\frac{c^2}{b^2}+\frac{b^2}{a^2}+2(\frac{a}{b}+\frac{c}{a}+\frac{b}{c})\geq\frac{3(a^2+b^2+c^2)}{\sqrt[3]{(abc)^2}}$

Áp dụng BĐT AM-GM ta có $\frac{a^2}{c^2}+\frac{a}{b}+\frac{a}{b}\geq3\sqrt[3]{\frac{a^4}{b^2c^2}}=\frac{3a^2}{\sqrt[3]{(abc)^2}}$

Tương tự với các bđt còn lại, sau đó cộng lại ta có đpcm.

  • Trở lại bài toán, áp dụng bđt phụ trên thì ta cần chứng minh $\frac{\sqrt{3(a^2+b^2+c^2)}}{\sqrt[3]{abc}}+\sqrt[3]{abc}\geq\frac{10}{9(a^2+b^2+c^2)}$

Áp dụng bđt AM-GM ta có

$\frac{\sqrt{3(a^2+b^2+c^2)}}{\sqrt[3]{abc}}+\sqrt[3]{abc}=\frac{\sqrt{3(a^2+b^2+c^2)}}{\sqrt[3]{abc}}+9\sqrt[3]{abc}-8\sqrt[3]{abc}\geq2\sqrt{9\sqrt{3(a^2+b^2+c^2)}}-\frac{8}{3}(a+b+c)=6\sqrt[4]{3(a^2+b^2+c^2)}-\frac{8}{3}$

Đặt $t=\sqrt[4]{3(a^2+b^2+c^2)}\geq\sqrt[4]{(a+b+c)^2}=1$

Ta chỉ cần chứng minh $6t-\frac{8}{3}\geq\frac{10}{3t^4}\Leftrightarrow t^4(9t-4)\geq5$ (đúng do $t\geq1$ )

Vậy ta có đpcm. ~O)

 




#723698 Một số bài toàn bất đẳng thức mình cần được giúp đỡ!

Đã gửi bởi toanND on 12-07-2019 - 16:52 trong Bất đẳng thức và cực trị

$\boxed{16}$

Áp dụng BĐT Cauchy Schwarz ta có

$\frac{a}{4b^2+1}+\frac{b}{4c^2+1}+\frac{c}{4a^2+1}=\frac{a^3}{4a^2b^2+a^2}+\frac{b^3}{4b^2c^2+c^2}+\frac{c^3}{4c^2a^2+c^2}\geq \frac{(a\sqrt{a}+b\sqrt{b}+c\sqrt{c})^2}{4(a^2b^2+b^2c^2+c^2a^2)+a^2+b^2+c^2}$

Ta chỉ cần chứng minh 

$4(a^2b^2+b^2c^2+c^2a^2)+a^2+b^2+c^2\leq 1 =(a+b+c)^2$

$\Leftrightarrow ab(1-2ab)+bc(1-2bc)+ca(1-2ca)\geq0$

Lại có $ab>0$ , $ab\leq\frac{(a+b)^2}{4}<\frac{(a+b+c)^2}{4}$ $\Rightarrow ab<\frac{1}{4}\Rightarrow ab(1-2ab)>0$

Tương tự ta có đpcm

Dấu = không xảy ra




#723607 CM : $\frac{x^2y}{y} + \frac{y^2z...

Đã gửi bởi toanND on 08-07-2019 - 21:32 trong Bất đẳng thức và cực trị

Cho 0 < x ≤ y ≤ z. CM : $\frac{x^2y}{y} + \frac{y^2z}{x} + \frac{z^2x}{y} \geqslant x^2 + y^2 + z^2$

Hình như điều kiện phải là $x\geq y\geq z>0$. Đây là VMO 1991




#723525 hỏi về lục giác suy biến

Đã gửi bởi toanND on 05-07-2019 - 10:06 trong Hình học

Mình đang học về định lý Brianchon nhưng lại gặp khó khăn ở cách áp dụng nó cho lục giác suy biến. Ai có thể cho mình một số ví dụ về lục giác ngoại tiếp suy biến  được không? (kèm theo hình). Mình cảm ơn.




#723438 Cho các số thực dương a, b, c

Đã gửi bởi toanND on 02-07-2019 - 09:14 trong Bất đẳng thức và cực trị

Cho các số thực dương a, b, c. Chứng minh rằng 

$\frac{1}{(a+b)^2}+\frac{1}{(b+c)^2}+\frac{1}{(c+a)^2}\geq \frac{3\sqrt{3abc(a+b+c)}(a+b+c)^2}{4(ab+bc+ca)^3}$




#723414 Hê thức lượng

Đã gửi bởi toanND on 30-06-2019 - 16:49 trong Hình học

$\boxed{4}$

hethuc2.PNG

a. Ta có $BK^2=BA^2=BH.BC\Rightarrow \bigtriangleup BKH \sim \bigtriangleup BCK\Rightarrow \widehat{BKH}=\widehat{BCK}$

   Tương tự $\bigtriangleup DKL \sim \bigtriangleup DCK\Rightarrow \widehat{DKL}=\widehat{BCK}$

   Vậy $\widehat{BKH}=\widehat{DKL}$

b. Dựng hai đường tròn $(B,BK)$  $(D,DK)$. Gọi T là giao điểm thứ hai của hai đường tròn này.

    Gọi M, N lần lượt là giao điểm của TK với AC, BC.

    Dễ thấy AC là tiếp tuyến chung của (B,BK) và (D, DK)

    Ta có $MA^2=MK.MT=MI^2\Rightarrow MA=MI$. Lại có $MN\parallel AH\parallel IL$ (cùng vuông BC) nên NH = NL.

    Mặt khác $KN\perp HL$ nên tam giác HKL cân $\Rightarrow KH=KL$

c. Từ $\bigtriangleup DKL \sim\bigtriangleup DCK$ kết hợp với KH = KL ta có $\widehat{KHC}=\widehat{KLH}=\widehat{DKC}$

    Do đó $\bigtriangleup CKD\sim\bigtriangleup CHK\Rightarrow CK^2=CD.CH$. Lại có tứ giác AIDH nội tiếp nên $CD.CH=CI.CA$

    Vậy $CK^2=CA.CI$  ~O)

 




#723413 Hê thức lượng

Đã gửi bởi toanND on 30-06-2019 - 16:14 trong Hình học

Em cảm ơn anh toanND, em check lại bài 2 đề bài vẫn đúng ah

Ở câu b/ nếu $\angle BCA=90^0$ thì tam giác ABC có hai góc vuông à?




#723405 Hê thức lượng

Đã gửi bởi toanND on 29-06-2019 - 22:33 trong Hình học

e coi lại đề bài 2 xem 

nhimtom



#723404 Hê thức lượng

Đã gửi bởi toanND on 29-06-2019 - 22:28 trong Hình học

$\boxed{3}$

Kẻ EK, FI lần lượt vuông góc với AB, AC tại K, I.

Ta có $\frac{AK}{BK}=\frac{AK.AB}{BK.AB}=\frac{EA^2}{EB^2}=4=\frac{HC}{HB}$

theo định lý Thales đảo $\Rightarrow HK||AC$. Tương tự $HI\parallel AB$

$\Rightarrow AIHK$ là hình bình hành

Ta có $\bigtriangleup AEB\sim \bigtriangleup CFA \Rightarrow \measuredangle BAE=\measuredangle ACF=\measuredangle AFI\Rightarrow \bigtriangleup AEK\sim \bigtriangleup FAI$

$\Rightarrow \frac{EK}{AK}=\frac{AI}{FI}\Rightarrow \frac{EK}{AI}=\frac{AK}{FI}$. Mà $AK = HI,AI=HK$

$\Rightarrow \frac{EK}{HK}=\frac{HI}{IF}$. Lại có $\measuredangle EKH=\measuredangle BKH +90^0=\measuredangle HIC+90^0=\measuredangle HIF$

Do đó $\bigtriangleup EKH\sim\bigtriangleup HIF\Rightarrow\measuredangle HEK=\measuredangle IHF$

Vậy $\measuredangle EHF=\measuredangle EHK+\measuredangle KHI+\measuredangle IHF=(\measuredangle EHK+\measuredangle HEK)+\measuredangle BKH=180^0-\measuredangle EKH+\measuredangle BKH=180^0-90^0=90^0$

 




#723400 Hê thức lượng

Đã gửi bởi toanND on 29-06-2019 - 17:40 trong Hình học

$\boxed{\text{1}}$

hethuc1.PNG

a. Dễ thấy tứ giác KIHB nội tiếp $\rightarrow CI.CK=CH.CB=CA^2$

b. theo tính chất đối xứng thì $\measuredangle BDC=\measuredangle BAC=90^0;AC=DC$

Vì $\measuredangle BKC=\measuredangle BAC=\measuredangle BDC=90^0$ nên $B,K,A,C,D$ cùng thuộc một đường tròn

$\Rightarrow \measuredangle CKD=\measuredangle DAC=\measuredangle ADC=\measuredangle AKC$

ta có đpcm

 




#723109 bất đẳng thức

Đã gửi bởi toanND on 17-06-2019 - 12:51 trong Bất đẳng thức và cực trị

Áp dụng BĐT $b^2+bc+c^2\geq\frac{3}{4}(b+c)^2$

Ta có $15\geq3a^2+4(b^2+bc+c^2)\geq3[a^2+(b+c)^2]\geq\frac{3}{2}(a+b+c)^2$

$\Rightarrow (a+b+c)^2\leq10\Rightarrow-\sqrt{10}\leq a+b+c\leq \sqrt{10}$

Từ đó ta có min , max

e tự tìm dấu = nhé




#723098 bất đẳng thức

Đã gửi bởi toanND on 16-06-2019 - 22:23 trong Bất đẳng thức và cực trị

BÀI 3. Áp dụng BĐT Bunhiacopxki ta có $(a^2+b)(1+\frac{1}{b})\geq(a+1)^2$

Tương tự với các BĐT còn lại rồi nhân lại ta có $(a^2+b)(b^2+c)(c^2+a)\frac{(a+1)(b+1)(c+1)}{abc}\geq(a+1)^2(b+1)^2(c+1)^2$

$\Leftrightarrow (a^2+b)(b^2+c)(c^2+a)\geq abc(a+1)(b+1)(c+1)$

Dấu = xảy ra khi a =b =c 




#723048 bất đẳng thức

Đã gửi bởi toanND on 14-06-2019 - 16:37 trong Bất đẳng thức và cực trị

Ngưỡng mộ anh toanND quá, anh chỉ cho em cách học giỏi bđt với ạ

Kiếm sách với tài liệu mà đọc thôi e :ukliam2:




#723037 bất đẳng thức

Đã gửi bởi toanND on 14-06-2019 - 13:22 trong Bất đẳng thức và cực trị

Em cảm ơn anh toannd, bài 1 câu a anh có thể giải bằng cauchy dc k ah

Có thể dùng AM - GM (Cauchy) kiểu này : $\frac{1}{a}+\frac{1}{b}\geq\frac{4}{a+b}; \frac{1}{ab}\geq\frac{4}{(a+b)^2}$

Áp dụng hai BĐT trên, ta biến đổi biểu thức P như sau:

$P=(1+\frac{1}{a}+\frac{1}{b}+\frac{1}{ab})(1+\frac{1}{c}+\frac{1}{d}+\frac{1}{cd})\geq[1+\frac{4}{a+b}+\frac{4}{(a+b)^2}][1+\frac{4}{c+d}+\frac{4}{(c+d)^2}]=[(\frac{2}{a+b}+1)(\frac{2}{c+d}+1)] ^2 ]$

Đặt $A=[(\frac{2}{a+b}+1)(\frac{2}{c+d}+1)]^2$

$\Rightarrow A=[\frac{4}{(a+b)(c+d)}+\frac{2}{a+b}+\frac{2}{c+d}+1]^2\geq[\frac{16}{(a+b+c+d)^2}+\frac{8}{a+b+c+d}+1]^2=625$

$\Rightarrow P\geq A\geq625$  ~O)




#723026 bất đẳng thức

Đã gửi bởi toanND on 14-06-2019 - 08:50 trong Bất đẳng thức và cực trị

BÀI 1

a. Áp dụng BĐT Holder ta có: $P=(\frac{1}{a}+1)(\frac{1}{b}+1)(\frac{1}{c}+1)(\frac{1}{d}+1)\geq (\sqrt[4]{\frac{1}{abcd}}+1)^{4}$

Mặt khác theo BĐT AM-GM: $\sqrt[4]{abcd}\leq \frac{a+b+c+d}{4}=\frac{1}{4}$

$\Rightarrow P\geq(4+1)^{4}=625$

Vậy $minP=625$ khi $a=b=c=d= \frac{1}{4}$

b. Ta có $Q=\frac{a(b+c+d)}{\frac{a^{2}}{3}+b^{2}+\frac{a^{2}}{3}+c^{2}+\frac{a^{2}}{3}+d^{2}}\leq\frac{a(b+c+d)}{\frac{2}{\sqrt{3}}(ab+ac+ad)}=\frac{\sqrt{3}}{2}$

Vậy $maxQ = \frac{\sqrt{3}}{2}$ khi ..........

BÀI 2. Ý tưởng cũng giống bài 1b thôi e  :closedeyes:




#722984 Chứng minh rằng $\widehat{APB}=\widehat{APD...

Đã gửi bởi toanND on 12-06-2019 - 10:40 trong Hình học

$\boxed{\text{Bài toán}}$ [Bulgaria TST 2003] Cho tứ giác ABCD ngoại tiếp đường tròn (O). Hạ OP vuông góc AC. Chứng minh rằng $\widehat{APB}=\widehat{APD}$

Hình gửi kèm

Bulgaria TST 2003.PNG




#722981 Tìm GTNN của P

Đã gửi bởi toanND on 12-06-2019 - 08:32 trong Bất đẳng thức và cực trị

Đầu tiên ta có đẳng thức$a^{2}+1=a^{2}+ab+bc+ca=(a+b)(a+c)$

Tương tự: $b^{2}+1=(b+c)(b+a), c^{2}+1=(c+a)(c+b)$

Ta có $P=\sum \sqrt{\frac{(b^{2}+1)(a^{2}+1)}{c^{2}+1}}=\sum \sqrt{\frac{(b+c)(b+a)(a+b)(a+c)}{(c+a)(c+b)}}=\sum (a+b)=2(a+b+c)$

$\Rightarrow P\geq 2\sqrt{3(ab+bc+ca)}=2\sqrt{3}$

Dấu = xảy ra khi $a=b=c=\frac{\sqrt{3}}{3}$

Vậy $minP=2\sqrt{3}$ khi $a=b=c=\frac{\sqrt{3}}{3}$




#722942 Chứng minh rằng B, E, R thẳng hàng.

Đã gửi bởi toanND on 10-06-2019 - 21:25 trong Hình học

Anh thấy có nhiều cách giải mà, đâu chỉ mỗi Pascal đâu !!

attachicon.gifScreenshot from 2019-06-10 21-18-36.png

Tại em thấy bài này trong bài tập về định lý Pascal nên mới tìm cách giải bằng Pascal nhưng lại chưa ra. Nhưng cũng cảm ơn a.




#722939 Chứng minh rằng B, E, R thẳng hàng.

Đã gửi bởi toanND on 10-06-2019 - 21:16 trong Hình học

Có cách nào sử dụng định lý Pascal không nhỉ?




#722933 Chứng minh rằng B, E, R thẳng hàng.

Đã gửi bởi toanND on 10-06-2019 - 17:25 trong Hình học

$\boxed{\text{APMO 2013}}$ Cho tứ giác ABCD nội tiếp đường tròn (O). P nằm trên tia AC sao cho PB, PD tiếp xúc với (O). Tiếp tuyến của (O) tại C cắt PD tại Q, AD tại R. E là giao điểm thứ hai của AQ và (O). Chứng minh rằng B, E, R thẳng hàng.

Hình gửi kèm 

APMO 2013.PNG




#722880 Cho x, y, z là các số dương

Đã gửi bởi toanND on 09-06-2019 - 16:08 trong Bất đẳng thức và cực trị

Đặt $P=\sum \frac{x^{3}+y^{3}}{x+2y}$. Ta áp dụng BĐT AM-GM như sau:

$\frac{x^{3}}{x+2y}+\frac{x^{3}}{x+2y}+\frac{(x+2y)^{2}}{27}\geq 3\sqrt[3]{\frac{x^{6}(x+2y)^{2}}{27(x+2y)^{2}}}=x^{2}$

$\frac{y^{3}}{x+2y}+\frac{y^{3}}{x+2y}+\frac{(x+2y)^{2}}{27}\geq y^{2}$

Cộng vế theo vế hai BĐT trên $\Rightarrow \frac{2(x^{3}+y^{3})}{x+2y}+\frac{2}{27}(x+2y)^{2}\geq x^{2}+y^{2}$

Làm tương tự với các BĐT còn lại rồi cộng vế theo vế ta được:

$2P+\frac{2}{27}\sum (x+2y)^{2}\geq 2\sum x^{2}$

Khai triển, chuyển vế các thứ ta có $P\geq \frac{22}{9}-\frac{4}{27}(xy+yz+zx)\geq \frac{22}{9}-\frac{4}{27}(x^{2}+y^{2}+z^{2})=2$

Dấu = xảy ra khi x = y = z = 1